Day 3 A&E Flashcards

1
Q

A 65-year old man presents to the A&E department with a 5 hour history of palpitations.

He has a past medical history of Ischaemic Heart Disease, and he had an MI 2 years ago. His basic observations are as follows:

HR 150, RR 25, BP 125/80, T 37.3, SO2 96%

On examination, JVP was not visible and there was no peripheral oedema. He has palpable peripheral pulses.

Auscultation of his heart and lungs were normal. A portion of his ECG is shown below:

What is the most appropriate management for this patient?

A

IV Amiodarone

This patient has developed a monomorphic broad complex tachycardia on his ECG.

This is also known as a ventricular tachycardia (VT).

The most appropriate management of VT in a haemodynamically stable patient is IV Amiodarone.

  • NOT Vagal manoeuvres such as carotid sinus massage and Valsalva can be used as the first line management in stable patients with a narrow complex tachycardia, not broad complex tachycardia.
How well did you know this?
1
Not at all
2
3
4
5
Perfectly
2
Q

A 27-year-old male presents to A&E unconscious.

His basic observations; HR: 110, RR: 35, BP: 90/65 and Temp. 36.5

As part of his work up and ECG is taken.

What is the most likely explanation for the appearance of his ECG.

A

Acute pancreatitis

This patient’s ECG shows a polymorphic ventricular tachycardia, with QRS complexes ‘twisting’ along the baseline - this is an ECG of Torsades de Pointes. Torsades de Pointes is caused by lengthening of the QT interval (this refers to a QT interval >450ms).

There are many causes of Torsades de Pointes, one of which is hypocalcaemia, as hypocalcaemia causes prolongation of the QT interval.

Acute pancreatitis can cause hypocalcaemia and therefore, lead to Torsades de Pointes

How well did you know this?
1
Not at all
2
3
4
5
Perfectly
3
Q

What causes torsades de pointes? (3)

What is the treatment? (1)

A

hypocalcaemia, hypokalaemia, hypomagnesaemia,

IV magnesium sulfate

How well did you know this?
1
Not at all
2
3
4
5
Perfectly
4
Q

A 32 year old man is admitted to hospital following a road traffic accident. Initial assessment reveals extensive pelvic injury.

The patient is drowsy and speaking incoherently.

A set of observations are performed during initial resuscitation:

  • Heart rate 128 bpm
  • Blood pressure 83/68 mmHg
  • Respiratory rate 36 /min
  • Oxygen saturations 98% on room air
  • Temperature 36.5 °C

What stage of hypovolaemic shock is the patient most likely to be in?

A

Stage 3

The patient has all the features of stage 3 shock, confusion, tachycardia, hypotension, tachypnoea and reduced pulse pressure.

How well did you know this?
1
Not at all
2
3
4
5
Perfectly
5
Q

A 50-year-old lady presents to the A&E Department with a 1 day history of pleuritic chest pain. She has a past medical history of COPD. Her observations are as follows:

HR 88, RR 20, BP 120/85, T 37.0, SO2 89%

On examination, she is comfortable at rest. On palpation, there is no tracheal deviation or crepitus over the chest. On auscultation, there is decreased air entry over her right lung.

A chest X-ray is ordered and this is shown below:

What is the best management option for this patient?

A

Chest drain insertion

This patient has features of a secondary spontaneous right-sided pneumothorax, most likely secondary to her underlying COPD.

Patients who have a secondary pneumothorax who are symptomatic or have a pneumothorax measuring >2 cm from the chest rim on radiographs should be offered a chest drain.

How well did you know this?
1
Not at all
2
3
4
5
Perfectly
6
Q

A 23 year old lady presents to the Emergency Department (ED) having taking a paracetamol overdose. She reports taking 12 tablets over a couple of hours earlier that morning, with the first tablet taken 3 hours ago.

On assessment she is asymptomatic and clinically stable. Her weight is approximately 60kg.

Which of the following is the next best step in management?

A

Start treatment with N-acetylcysteine (NAC) immediately

This patient has taken a staggered overdose which is defined as being an overdose taken over > 1 hour.

In these cases, measuring serum paracetamol levels and plotting on a nomogram is not a reliable method to determine if treatment is required.

Patients will usually be given three bags of NAC over a period of 21 hours.

How well did you know this?
1
Not at all
2
3
4
5
Perfectly
7
Q

An 89-year-old man presents to A&E with abdominal pain.

The pain has worsened over the past two days and is associated with nausea, vomiting and diarrhoea.

He also reports feeling weak and lethargic.

His past medical history includes hypertension and congestive heart failure The abdomen is soft and non-tender.

Observations are as follows: respiratory rate 11/min, SpO2 95% on room air, pulse rate 130/min, blood pressure 123/87, temperature 37.2.

Baseline blood tests show a hypokalaemia (3.2 mmol/L) but are otherwise normal.

A 12-lead ECG is performed, which shows prolonged PR interval and a down-sloping ST-segment depression most evident in lead V6 and II.

Which of the following is the most likely cause of this patient’s presentation?

A

Digoxin toxicity

This patient most likely takes Digoxin for congestive heart failure. The abdominal pain, nausea and vomiting may be explained by gastroenteritis, or due to the effects of Digoxin toxicity itself.

Hypokalaemia (which can result from vomiting and diarrhoea) worsens Digoxin toxicity - allowing it to occur in therapeutic concentrations.

The downsloping ST segment is the characteristic ‘Salvador Dali’s moustache’ or reverse tick sign - this does not necessarily indicate toxicity, but is seen with Digoxin use.

Note, yellow discolouration of vision (xanthopsia) is a classic but rare sign of Digoxin toxicity.

How well did you know this?
1
Not at all
2
3
4
5
Perfectly
8
Q

A 75 year old man with a history of IVDU (now on methadone for the last 20 years) comes into the A&E after suspected food poisoning.

He feels sick and has vomited four times.

His family have all had similar symptoms earlier today after eating a salad at the local takeaway.

An admission ECG shows a QTc of 490ms.

He is given something for his nausea when he arrived at triage.

He suddenly collapses and is found to be pulseless.

What medication did he receive?

A

A serotonin 5HT3 antagonist, it is often used as an adjunct with chemotherapy to reduce nausea.

Importantly, it can be given PO/IM/IV making it useful when a patient is unable to keep oral foods down.

One should note that ondansetron can increase QT interval and put a patient at risk of torsades de pointes which is what happened in this case.

This patient is already on another drug notorious for prolonging QT interval – methadone.

How well did you know this?
1
Not at all
2
3
4
5
Perfectly
9
Q

A 57 year old farmer presents with difficulty breathing, diarrhoea, urinary frequency and muscle spasms.

On examination his eyes are watering and he appears sweaty.

He is also bradycardic.

What is the most likely cause of his presentation?

A

Organophosphates

Organophosphates (found in pesticides) cause over-activity of the cholinergic system giving the symptoms described in this scenario.

How well did you know this?
1
Not at all
2
3
4
5
Perfectly
10
Q

A 72-year-old woman presents to the emergency department with chest pain and palpitations.

On examination, she has a rapid, regular pulse and bibasal inspiratory crackles.

Her observations are below:

  • HR 200
  • BP 92/55
  • RR 25
  • SaO2 95%
  • Afebrile

ECG shows a regular QRS rate of 150bpm and QRS duration > 120ms (>3 small squares).

Having completed an ABCDE assessment and established IV access, what is the single best next step?

A

Synchronised DC cardioversion

This patient has monomorphic VT and is displaying adverse signs (chest pain, indicating myocardial ischaemia, haemodynamic instability and pulmonary oedema).

Therefore, she is clinically unstable and requires immediate electrical cardioversion.

How well did you know this?
1
Not at all
2
3
4
5
Perfectly
11
Q

What is a normal QRS width?

what can causes an elongated QRS?

A

Normal QRS width is 70-100 ms

hypokalaemia

How well did you know this?
1
Not at all
2
3
4
5
Perfectly
12
Q

Treatments for

VT (2)

SVT (2)

A

VT

  • Amiodarone 300mg IV when the patient is clinically stable
  • synchronised dc cardioversion

SVT

  • vasovagal maneouvres when the patient is clinically stable
  • adenosine 6mg iv
How well did you know this?
1
Not at all
2
3
4
5
Perfectly
13
Q

An 83 year old man with a past medical history of heart failure presents with shortness of breath, raised JVP, and peripheral oedema.

Auscultation of the lungs reveals bibasal crackles.

Observations are O2 saturation of 94%, respiratory rate of 22, HR 106, BP 82/54, and temperature of 37.1 degrees Celsius.

An echocardiogram performed 3 weeks ago identified an ejection fraction of 20%.

Multiple doses of IV furosemide are given but no improvement is seen.

What is the which of the following is the best next step in management?

A

IV dobutamine

Intravenous inotropes (e.g. dobutamine, milrinone) may sometimes be necessary in managing patients with a severely reduced LV systolic function.

Inotropes increase the contractility of the heart.

These measures are done to maintain systemic perfusion to the end-organs.

How well did you know this?
1
Not at all
2
3
4
5
Perfectly
14
Q

A 22 year old man is brought into the Emergency Department (ED) following an asthma attack.

He is given nebulised salbutamol and ipratropium inhalers, IV hydrocortisone and IV magnesium sulphate.

Despite this, he remains short of breath and auscultation of the chest reveals a wide-spread expiratory wheeze.

An arterial blood gas (ABG) is performed on 15L of oxygen and shows:

  • pH: 7.32 (7.35-7.45)
  • PaO2: 7.6 kPa (11-15 kPa)
  • PaCO2: 6.3 kPa (4.5-6 kPa)
  • Bicarbonate: 27 mmol/L (22-28 mmol/L)

Which of the following is the next best step in management?

  • Bilevel positive pressure ventilation (BiPAP)
  • IV salbutamol
  • Mechanical ventilation
  • IV aminophylline
  • Continuous positive pressure ventilation (CPAP)
A

Mechanical ventilation

This patient has near-fatal asthma which is characterised by raised PaCO2 levels.

Clinically the patient is likely to be exhausted and unable to sustain ventilatory effort thereby leading to Type 2 respiratory failure as shown on the ABG.

The most effective method of managing this would be to intubate the patient and start mechanical ventilation.

How well did you know this?
1
Not at all
2
3
4
5
Perfectly
15
Q

A 58-year old man presents to the Emergency Department with nausea and vomiting for the past two hours.

He is known to the alcohol liaison service and his past medical history includes severe depression.

His wife states he has ingested a bottle of anti-freeze.

Examination is unremarkable and observations are stable.

An arterial blood gas (ABG) demonstrates a metabolic acidosis.

What is the most appropriate management in this case?

A

Fomepizole

Fomepizole inhibits alcohol dehydrogenase.

At a sufficiently high concentration, ethanol saturates alcohol dehydrogenase, preventing it from acting on ethylene glycol, thus allowing the latter to be excreted unchanged by the kidneys.

Historically, this has been done with intoxicating doses of ethanol. However, ethanol therapy is complicated by its own toxicity.

Fomepizole inhibits alcohol dehydrogenase without producing serious adverse effects.

How well did you know this?
1
Not at all
2
3
4
5
Perfectly
16
Q

A 20-year-old gentleman is brought into the Emergency Department.

He is a known epileptic and is taking valproate regularly.

He started seizing about 5 minutes ago and is still currently jerking uncontrollably.

His basic observations are as follows: HR 100, RR 12, BP 140/90, T 35.8, SO2 92% RA.

He is immediately put in the recovery position, started on high flow oxygen and an oropharyngeal airway adjunct is inserted.

His blood glucose level is 6.2 mmol/L. After about 15 minutes, he is still seizing.

2 doses of IV Lorazepam 4mg had been administered but it did not seem to help.

His SO2 has improved to 98% on room air.

What is next most appropriate treatment option for this patient?

A

IV infusion of Phenytoin

This patient is in status epilepticus that has persisted for 15 minutes and failed to improve on 2 doses of IV Lorazepam.

A second, more longer-acting anticonvulsant is needed at this stage, and guidelines recommend an infusion of Phenytoin.

How well did you know this?
1
Not at all
2
3
4
5
Perfectly
17
Q

The surgical foundation doctor is called to assess a 32 year old male on a surgical ward.

On arrival the patient is comatose and appears to be making a poor respiratory effort; there is no apparent sign of airway compromise.

The patient’s oxygen saturations are 78%, the respiratory rate is 7 breaths per minute, and heart rate is 121 beats per minute.

What is the next most appropriate action?

A

Commence manual ventilation with a bag-valve mask

This patient has signs of significant respiratory compromise with low oxygen saturations and a low respiratory rate.

Given the respiratory rate of <10 and associated hypoxia, this patient needs manual ventilation, connecting the bag-valve mask to 15L/min oxygen.

Given a nurse is present, they can assist in calling for help or pulling the emergency buzzer, while the clinician starts ventilation.

How well did you know this?
1
Not at all
2
3
4
5
Perfectly
18
Q

A 60-year old man presents to the Emergency department with palpitations and dizziness.

His past medical history includes ischaemic heart disease.

On examination he is sweating and has bibasal lung crepitations.

His blood pressure is 78/52mmHg, respiratory rate 18, Sp02 98% (room air) and temperature 37.4.

His ECG shows a ventricular tachycardia.

What is the most appropriate management in this case?

A

Cardioversion

As this patient has a ventricular tachycardia (VT) as well as signs of haemodynamic compromise (hypotension) the first-line management is cardioversion.

VT is a broad complex tachycardia originating from a ventricular ectopic focus.

If treated rapidly, VT generally has a favourable short-term outcome.

VF or pulseless VT is treated by unsynchronised defibrillation; whereas other VTs can be treated with synchronised cardioversion.

Amiodarone is used in patients with VT; however, it is not first-line in haemodynamically unstable VT.

How well did you know this?
1
Not at all
2
3
4
5
Perfectly
19
Q

When should Implantable cardioverter defibrillators (ICDs) be used?

(2)

A

ICDs should be considered in patients who have sustained VT causing syncope, sustained VT with an ejection fraction less than 35% or a previous cardiac arrest due to VT or ventricular fibrillation (VF).

ICDs can also be used in myocardial infarctions which are complicated by non-sustained VT.

How well did you know this?
1
Not at all
2
3
4
5
Perfectly
20
Q

A 58-year old man is rushed into the Emergency Department via ambulance. He was found unresponsive by a passer-by. There is nobody available to provide any further information.

The ambulance crew provide the following ECG

What is the most likely diagnosis?

How can he be manged?

A

Ventricular Fibrillation

Ventricular fibrillation (VF) is an important shockable cardiac arrest rhythm.

This rapid and irregular electrical activity means the ventricles are unable to contract in a synchronised manner, which results in an immediate loss of cardiac output.

Management of VF and pulseless VT

Defibrillation and CPR are the mainstays of treatment. However if persistent, Amiodarone 300mg IV (same dose used to treat monomorphic VT) and Adrenaline 1mg IV (1:10,000) can be given after the third shock has been delivered.

Amiodarone is given as a one-off dose. However Adrenaline may be repeated every other cycle following a shock (i.e. cycles 3, 5, 7 etc.)

How well did you know this?
1
Not at all
2
3
4
5
Perfectly
21
Q

The medical foundation doctor has been called urgently to a general medical ward to assess a 45 year old gentleman who has become unresponsive.

The student nurse at the bedside does not know any background or history about the patient.

On A to E assessment, the patient is maintaining his airway, and no abnormality is noted on breathing or circulation assessment.

On assessing disability, the GCS is 4/15, pupils are equal and reactive to light, and bedside blood glucose is 2.3mmol/L.

There is an IV cannula in situ.

What is the single best initial treatment?

A

100ml 20% glucose IV over 10 minutes

This patient has severe hypoglycaemia causing his unresponsiveness.

He needs to be given 15 - 20g of rapid-acting carbohydrate.

Given this patient is unconscious, IV treatment is indicated. 100ml of 20% glucose corresponds to 20g of glucose, and guidance suggests this is given over 10 mins (although practically it may be given as a slow IV push via a syringe).

How well did you know this?
1
Not at all
2
3
4
5
Perfectly
22
Q

A 20-year-old man with no significant past medical history is brought into A&E by ambulance after a collapse.

He was playing football, suddenly fell to the floor, and was unconscious for around 5 minutes.

He reports palpitations prior to the collapse, but no shortness of breath, dizziness, chest pain, or visual symptoms.

There was no abnormal limb jerking while he was on the floor, and no post-ictal state. Neurological examination is unremarkable.

A 12-lead ECG shows a regular, narrow complex tachycardia, with short PR interval and slurred upstrokes on the QRS complexes.

What is the most likely diagnosis?

A

Wolff-Parkinson-White syndrome

Wolff-Parkinson-White syndrome is a type of supraventricular tachycardia caused by a congenital accessory conduction pathway in the heart.

The classical sign on ECG are short PR intervals and delta waves (slurred upstroke in the QRS complex).

Treatment is by ablation of the accessory pathway.

How well did you know this?
1
Not at all
2
3
4
5
Perfectly
23
Q

Features on ECG in WPW

(4)

A

Delta waves (slurred upstroke in the QRS)

Short PR interval (<120ms)

Broad QRS

If a re-entrant circuit has developed the ECG will show a narrow complex tachycardia

How well did you know this?
1
Not at all
2
3
4
5
Perfectly
24
Q

A 35-year-old East Asian lady presents to the Emergency Department complaining of a headache, which came on quite suddenly last evening.

She also feels very nauseous and noticed her left eye becoming quite red.

Apart from having migraines as a teenager, she is otherwise fit and healthy with no significant past medical history.

She recently started taking amitriptyline as she had trouble sleeping.

She has no drug allergies.

Her basic observations are as follows:

HR 100, RR 20, T 37.0, BP 130/80, SO2 98% RA.

Give her underlying diagnosis

Which of the following is the best initial treatment for her?

A

Acetazolamide and Timolol

This lady has acute angle closure glaucoma.

Risk factors include being female, Asian and the use of certain medications including those with antimuscarinic properties, such as amitriptyline.

Patients with acute angle closure glaucoma complain of a sudden headache, nausea and loss of vision.

Symptoms may worsen at night.

The initial management includes administering IV Acetazolamide and a topical beta-blocker such as Timolol.

An urgent Ophthalmology referral should be made.

How well did you know this?
1
Not at all
2
3
4
5
Perfectly
25
Q

A 28-year-old man with a history of Type 1 Diabetes is found collapsed on the floor at a bus stop.

A witness reports that preceding the blackout, the man appeared agitated and confused.

On initial assessment, the patient has a GCS of 7 and his blood glucose level is 1.7 mmol/L (4-7 mmol/L).

IV access is obtained.

Which of the following is the next best step in management?

A

100ml of 20% glucose IV

This patient is displaying neuroglycopaenic symptoms of hypoglycaemia which typically develops when the blood glucose level is

  • less than 2.8 mmol/L.

These symptoms include confusion, drowsiness, seizures, slurred speech and loss of consciousness.

As this patient has a severely reduced level of consciousness, it is important not to put any glucose replacement in his mouth as this carries the risk of choking and aspiration.

Instead, glucose should be given IV at a concentration of either 20% or 10%.

How well did you know this?
1
Not at all
2
3
4
5
Perfectly
26
Q

A 45-year-old man is brought to the emergency department by his partner as she is concerned by his ‘overnight’ behaviour change. She states that he has been unwell with the ‘flu’ for several days, with fever, muscles aches and a sore throat. However last night, he became verbally abusive and agitated which is out of character for him. He does not have any ongoing health issues to her knowledge.

On examination, the patient appears confused and drowsy, and is disoriented to place and time. He is not compliant with neurological examination, but appears to shy away from the light, clutching his head. There is an erythematous rash with overlying scale over the forehead, nose and nasolabial folds, and a white coating over the tongue extending to the oropharynx. His partner states that the scaly rash appears everytime he gets the ‘flu’.

His observations are below:

  • HR 105
  • RR 21
  • BP 95/55
  • SaO2 99% RA
  • T 38.3

Give the likely causes of his symptoms?

What is the single best empirical treatment?

A

Aciclovir and Cefotaxime

This man is very ill with signs of an intracranial infection and a worrying drop in GCS.

The signs of meningeal irritation with fever and behaviour change are highly suggestive of meningoencephalitis.

Given the history of repeated seborrheic dermatitis during intercurrent illness and current oropharyngeal candidiasis, it is likely an opportunistic infection secondary to HIV.

Aciclovir covers for HSV-1 and HSV-2.

Cefotaxime is a 3rd generation cephalosporin and covers for most causes of bacterial meningitis.

How well did you know this?
1
Not at all
2
3
4
5
Perfectly
27
Q

A 24-year-old woman is brought into A&E by her friends with agitation and confusion.

On examination, she is shivering and tremulous, with hyperreflexia of her lower limbs.

Neurological examination is otherwise normal, and pupils are equal and reactive to light.

Observations are as follows: respiratory rate 12/min, SpO2 95% on room air, pulse rate 129/min, blood pressure 134/89, temperature 38.1.

She suffers from depression for which she is receiving treatment from her GP.

Which is the most likely diagnosis?

A

Serotonin syndrome

This patient has presented with features of serotonin syndrome, which can occur in patients taking therapeutic doses of serotonin reuptake inhibitors (SSRIs).

This syndrome is often confused with neuroleptic malignant syndrome, but the clinical features are distinct.

Clinical features of serotonin syndrome can be split into neurological and autonomic.

Neurological features are of altered mental state, tremor, ataxia and hyperreflexia.

Autonomic features include

  • tachycardia
  • hypertension
  • diarrhoea
  • hyperthermia
How well did you know this?
1
Not at all
2
3
4
5
Perfectly
28
Q

What is Neuroleptic malignant syndrome?

What is the predominant symptom?

A

This is often confused with serotonin syndrome.

It is caused by antipsychotic medications or sudden reduction in dopaminergic drugs (e.g. for Parkinson’s disease).

It can present with similar autonomic features, but the predominant neuromuscular feature is of rigidity (compared to tremor and hyperreflexia in serotonin syndrome).

How well did you know this?
1
Not at all
2
3
4
5
Perfectly
29
Q

A 75 year old man is reviewed by the cardiology registrar in resus.

He has a past medical history of atrial fibrillation and has been taking bisoprolol and amiodarone.

His heart rate fluctuates with a maximum rate of 150bpm.

The rest of the observations are within normal limits and electrolytes within normal range.

His ECG is reproduced below:

What is the next best initial step in management? (3)

A

IV Magnesium Sulphate and stop any causative medication

The patient’s ECG is consistent with Torsades de pointes a polymorphic ventricular tachycardia associated with prolonged QT.

It must be treated urgently due to the risk of unstable VT or VF leading to sudden cardiac death.

If the patient is haemodynamically unstable, then cardioversion is indicated

How well did you know this?
1
Not at all
2
3
4
5
Perfectly
30
Q

A 75-year-old gentleman presents to the Emergency Department with a 2-hour history of severe crushing central chest pain.

He has a past medical history of HTN, DM and IHD. His basic observations are as follows:

HR 100, RR 25, BP 150/95, T 37.3, SO2 97% RA.

An ECG is performed and it is shown below:

What is the most appropriate definitive treatment for this patient?

A

PCI

The ECG shows ST elevation in leads aVL, V2 to V5 with reciprocal changes in II, III, aVF.

This is diagnostic of a STEMI.

The definitive treatment for this is a PCI, involving inserting a stent to reperfuse the affected myocardium.

How well did you know this?
1
Not at all
2
3
4
5
Perfectly
31
Q

A 78 year old woman presents to A&E with haematemesis.

She has been having epigastric pain intermittently over the last 3 weeks, especially a few hours after eating.

She has a past medical history of osteoarthritis and hypertension.

She takes amlodipine and over-the-counter ibuprofen.

Which of the following is the definitive treatment for this patient?

A

Endoscopic clipping

The history of an upper GI bleed in a patient who has NSAID exposure, especially without concurrent protection with acid suppression, is highly suggestive of a bleeding peptic ulcer.

Endoscopy is the method of choice for managing bleeding from a peptic ulcer. Therapeutic measures include clipping, thermal coagulation, fibrin, or thrombin.

How well did you know this?
1
Not at all
2
3
4
5
Perfectly
32
Q

A 38-year-old woman presents to A&E with a two-day history of abdominal pain.

The pain is severe and most prominent in the epigastric region, with radiation to the back.

She reports nausea and several episodes of vomiting.

She has no significant past medical history but admits to regularly drinking at least 30 units of alcohol a week.

Observations are as follows:

  • respiratory rate 10/min,
  • SpO2 95% on room air,
  • pulse rate 130/min,
  • blood pressure 108/68,
  • temperature 37.9

A 12-lead ECG is performed which shows sinus tachycardia.

What is the most likely diagnosis?

A

Acute pancreatitis

Epigastric pain radiating to the back is characteristic of acute pancreatitis.

Patients may also describe that the pain is relieved by leaning forwards.

Gallstones and alcohol are the two biggest causes of pancreatitis, accounting for over 70% of cases.

Serum amylase and lipase will aid diagnosis.

How well did you know this?
1
Not at all
2
3
4
5
Perfectly
33
Q

An 82-year-old gentleman is brought into the Accident and Emergency department by ambulance acutely distressed with difficulty breathing and coughing up frothy pink sputum.

He has had two previous myocardial infarctions.

His observations are taken (see below).

On examination, he looks unwell.

He is sweaty and there are fine inspiratory crepitations throughout the lung fields.

  • Temperature: 37.2
  • Oxygen saturations: 97% on room air
  • Heart Rate: 87
  • Respiratory rate: 24
  • Blood pressure: 168/98

What is the next most immediate appropriate action to take?

A

Sit the patient upright

The underlying diagnosis is pulmonary oedema. This patient will likely need multiple investigations including ECG, chest X-ray and blood tests.

However, the most immediate action that can be performed is to sit the patient upright and improve oxygenation.

This patient may well end up receiving furosemide for management of pulmonary oedema.

However, in the first instance sitting the patient up may provide immediate symptomatic benefit.

How well did you know this?
1
Not at all
2
3
4
5
Perfectly
34
Q

A 72-year old is admitted to the Emergency Department with a 4-day history of lethargy and palpitations.

His past medical history includes hypertension but he is otherwise well and relatively active.

On examination his pulse is irregularly irregular.

His pulse rate is 118 bpm, respiratory rate 16 and blood pressure 132/76mmHg.

His ECG confirms Atrial Fibrillation (AF).

What is the most appropriate medication for first-line management?

A

Bisoprolol

As the patient is over the age of 65 and it is more than 48 hours after onset of symptoms, rate control is preferred over rhythm control.

Rate control options include beta-blocker (e.g. bisoprolol), a non-dihydropyridine calcium channel block (e.g. diltiazem or verapamil) or digoxin.

It is important to check for a history of asthma, as brittle asthma is a contraindication for beta-blockers due to the potential for bronchospasm.

How well did you know this?
1
Not at all
2
3
4
5
Perfectly
35
Q

A 28-year old gentleman presents to the A&E Department with a 4 hour history of diffuse abdominal pain, vomiting and lethargy.

His past medical history includes Type 1 DM, asthma and eczema.

He takes regular insulin, inhaled corticosteroids and PRN salbutamol.

He has no drug allergies.

On examination, he appears very lethargic.

His capillary refill time is 4 seconds.

His observations are as follows:

T: 37.3, HR: 105, RR: 25, O2: 98%, BP: 105/70

You quickly do an ABG and it shows the following:

  • pH: 7.22 (7.35 - 7.45)
  • PO2: 11.5 kPa (10 - 15)
  • PCO2: 4.3 kPa (4.5 - 6)
  • HCO3: 15 mmol/l (22 - 26)
  • Na: 148 mmol/l (135 - 145)
  • K: 5.0 mmol/l (4.5 - 5.0)
  • Lac: 1.0 mmol/l (0.5 - 1.0)
  • Glucose: 25 mmol/l

A urine dip shows the following:

  • Leucocytes NIL
  • Nitrites NIL
  • Protein +
  • Blood NIL
  • Ketones 3+

Which of the following is the best option for the initial management of this patient?

A

1L of 0.9% NaCl IV STAT

This patient appears severely dehydrated.

He has

  • vomiting
  • appears lethargic
  • has a capillary refill time of 4 seconds
  • a dropping BP
  • raised sodium level

He would need intravenous fluids as soon as possible, hence this is the right answer.

How well did you know this?
1
Not at all
2
3
4
5
Perfectly
36
Q

Clinical signs of dehydration

(5)

A

vomiting

appears lethargic

has a capillary refill time of 4 seconds

a dropping BP

raised sodium level

How well did you know this?
1
Not at all
2
3
4
5
Perfectly
37
Q

You are crash-bleeped to attend to a patient on AMU.

Other members of the team have arrived and CPR has already been commenced.

The defibrillator pads have just been attached and the ECG shows that the patient is in sinus rhythm with a ventricular rate of 65 bpm.

The carotid pulse is not palpable at the rhythm check.

What is the single next best step?

A

Recommence CPR

This patient has pulseless electrical activity (PEA).

This falls under the ‘non-shockable rhythm’ branch of the adult advanced life support algorithm.

It is important to minimise interruptions to CPR, which provides vital organ perfusion.

How well did you know this?
1
Not at all
2
3
4
5
Perfectly
38
Q

A 4 year old girl has severe difficulty breathing. She is drooling and appears unwell and in respiratory distress.

She has no relevant past medical history, although her mother is against vaccinations.

Her temperature measures 39.

What is the most likely microbiological cause of this patient’s condition?

A

Haemophilus influenzae B

This patient has acute epiglottitis, which is most frequently caused by bacterial infection of the soft tissue structures that are anatomically superior to the glottis.

Haemophilus influenzae B (Hib) is the most common cause of epiglottitis in non-vaccinated children.

How well did you know this?
1
Not at all
2
3
4
5
Perfectly
39
Q

A 19-year-old lady is brought into the Emergency Department (ED).

She reports worsening abdominal pain and vomiting over the last 3 days.

Additionally, she notes feeling more weak and drowsy over the last 24 hours.

Investigations reveal a random blood glucose level of 24 mmol/L (4-11 mmol/L) and a serum ketone level of 3 mmol/L (<0.1 mmol/L).

Which of the following arterial blood gas (ABG) patterns is most commonly seen in patients presenting with diabetic ketoacidosis?

A

Raised anion gap metabolic acidosis

The anion gap is the difference between measured cations and measured anions in the serum.

It is calculated by the following equation:

  • ([Na+] + [K+]) – ([Cl-] + [HCO3])

The normal range is approximately between 10-20 mmol/L.

In a raised anion gap metabolic acidosis, there is either endogenous or exogenous acid that leads to a decrease in the bicarbonate (in order to buffer the acid) thereby increasing the anion gap.

In DKA, the ketones are the source of the acidosis.

Other important causes of a raised anion gap metabolic acidosis include uraemia (in renal failure), lactic acidosis (e.g in sepsis) and ingestion of drugs such as aspirin, methanol and ethylene glycol.

How well did you know this?
1
Not at all
2
3
4
5
Perfectly
40
Q

A 45 year old male patient presents to the emergency department with palpitations and severe malaise.

He has a past medical history of unipolar depression, for which he is takes amitriptyline.

He has recently been treated for a lower respiratory tract infection.

Electrocardiogram reveals polymorphic wide complex tachycardia.

The patient is haemodynamically stable.

Give the likely diagnosis

Which of the management options is most appropriate?

A

Intravenous magnesium sulfate

The patient presents with torsades de pointes (TdP), a polymorphic ventricular tachycardia that has occurs due to QT prolongation (in this case most likely secondary to co-prescription of a tricyclic antidepressant with a macrolide).

Treatment is with intravenous magnesium sulphate, withdrawal of offending drugs, and correction of electrolyte abnormalities.

How well did you know this?
1
Not at all
2
3
4
5
Perfectly
41
Q

A 74-year-old woman presents to the Emergency Department by ambulance.

She was found at home alone by her neighbour with an empty bottle of medication next to her.

She appears to be confused and is complaining of dizziness and nausea.

She also complains of blurred vision with a yellow discolouration.

Her past medical history includes anxiety, depression, hypertension and atrial fibrillation.

Her observations include a heart rate of 30bpm, blood pressure 74/52mmHg and Sp02 95% (on room air).

The ECG reveals reverse tick ST depression with first degree heart block.

What is the diagnosis?

What is the first-line treatment?

What is the next most effective treatment for this patient?

A

Digoxin overdose

First line is atropine

Intravenous digoxin-specific antibody (Digibind)

The ECG reveals reserve tick ST depression with first degree heart block which is the classic effect seen in digoxin overdose.

This patient is experiencing severe digoxin toxicity (the presence of arrhythmia and xanthopsia - yellow vision, haemodynamically unstable) following an overdose of digoxin tablets with suicidal intention.

Digoxin-specific antibody (Digibind) is reserved for the reversal of life-threatening overdoses.

Digibind binds to molecules of digoxin, making them unavailable for binding at their site of action on cells in the body.

The fragment-digoxin complex accumulates in the blood, from which it is excreted by the kidney.

How well did you know this?
1
Not at all
2
3
4
5
Perfectly
42
Q

A 60-year old woman presents to the Emergency Department with shortness of breath.

Her past medical history includes angina.

On examination she is clammy.

Her blood pressure is 94/54mmHg.

Her ECG shows a sinus bradycardia of 38bpm.

What is the first initial step in management?

A

IV Atropine

The Resuscitation Council (UK) guidelines emphasise that the management of bradycardia depends on haemodynamic compromise and identifying the potential risk of asystole.

The factors indicating haemodynamic compromise (and therefore the need for treatment) are shock, syncope, myocardial ischaemia and heart failure.

IV Atropine 500 micrograms is given, and can be repeated every 3–5 min to a total of 3 mg.

How well did you know this?
1
Not at all
2
3
4
5
Perfectly
43
Q

How does IV Atropine work?

(2)

A

The most important therapeutic action of atropine is the inhibition of smooth muscle and glands innervated by postganglionic cholinergic nerves

It also dilates the pupils

How well did you know this?
1
Not at all
2
3
4
5
Perfectly
44
Q

A 32 year old male patient is rescued from a house fire.

He is displaying disorientation, nausea and lethargy.

Oxygen saturations are 100% on room air.

What is the likely COHb % in his blood?

A

20-40%

Carbon monoxide poisoning is a significant complication of exposure to fires, particularly if the fire is within a closed space (e.g. rooms of a house). It can present in non-specific ways and its signs and symptoms depend on the level of carboxylation of the patient’s haemoglobin. At this level of COHb lethargy, nausea and disorientation are common presentation. There is reduced oxygen available for cytochrome function which results in abnormal cellular functioning and occasional encephalopathy. Patients can therefore seem disoriented, confused and nauseated and to their detriment be dismissed as intoxicated.

45
Q

A 68-year old man presents to the Emergency Department shortness of breath.

He states he has recently had surgery to his bowel.

He denies any chest pain.

His past medical history includes COPD, hypertension and stroke.

His observations include a heart rate of 102bpm, blood pressure 112/74, respiratory rate 24/minute and Sp02 85% (on room air).

Clinical examination is unremarkable.

An ECG is performed which shows sinus tachycardia.

Which of the following is the best test to confirm the most likely diagnosis?

A

Computed tomographic pulmonary angiography (CTPA)

This patient has recently had surgery and is presenting with shortness of breath and tachycardia.

The most likely diagnosis is a pulmonary embolism (PE).

The ECG most commonly demonstrates a sinus tachycardia, although the S1Q3T3 pattern is characteristic (S-wave in lead I, Q-wave and inverted T-wave in lead III).

A CTPA will produce images of the pulmonary arteries, which are the blood vessels from the heart to the lungs.

This is used to identify any blockages in these vessels.

46
Q

A 35-year old man presents to the A&E Department with a history of increasing confusion over the past 2 hours.

He also complains of a bad headache and is feeling nauseous.

He had just returned from a ski holiday yesterday.

While skiing, his family recalls an incident where he fell over and hit his head against some rocks.

However, he appeared fine and walked away from that accident without any problems.

Currently, his basic observations are as follows:

HR 67, RR 30, T 36.7, SO2 94%, BP 150/95

What is the most likely cause of his current presentation?

A

Damage to middle meningeal vessels

Damage to the middle meningeal vessels would result in an Extradural haemorrhage (EDH).

Patients with an EDH usually experience a lucid interval after a recent traumatic incident.

This refers to a period where there is temporary improvement in a patient’s condition following injury, after which the condition deteriorates.

This patient has a history of a lucid interval following his ski accident. A few days later, he has started to deteriorate, showing signs of raised intracranial pressure (ICP), probably resulting from an extension and worsening of the bleed in his brain.

47
Q

A 78-year-old woman presents to A&E with shortness of breath.

She had recently been discharged from hospital following an elective total right hip replacement.

She reports no other symptoms such as chest pain, palpitations, or cough.

On examination, the right hip is tender over the surgical site, and there is swelling and tenderness of the right calf.

The chest is clear on auscultation and trachea central.

Observations are as follows:

  • respiratory rate 20/min
  • SpO2 89% on room air
  • pulse rate 121/min
  • blood pressure 132/98
  • temperature 37.1

A 12-lead ECG is performed which shows sinus tachycardia.

Which of the following is the most likely diagnosis?

A

Pulmonary embolism

Pulmonary embolism (PE) must always be considered in patients with unexplained hypoxia and/or breathlessness.

The swollen and tender right calf is most likely to be deep vein thrombosis, which makes a diagnosis of PE much more likely.

The major risk factor in this patient is the recent hip surgery. Sinus tachycardia is the most common ECG finding in PE.

48
Q

Features of digoxin toxicity

(4)

A

generally unwell, lethargy, nausea & vomiting, anorexia, confusion, yellow-green vision

arrhythmias (e.g. AV block, bradycardia)

gynaecomastia

yellow vision

49
Q

A 67-year-old man presents to A&E with confusion, fever and shortness of breath.

A full assessment is performed, with observation as follows:

  • respiratory rate 26/min,
  • SpO2 94% on high flow oxygen,
  • pulse rate 136/min,
  • blood pressure 92/61,
  • temperature 38.3.

While sepsis six is being initiated, it is noted that there is blood oozing from both cannula sites in the antecubital fossa.

A coagulation screen is as follows: platelet count low, fibrinogen low, prothrombin time (PT) prolonged, activated partial thromboplastin time (APTT) normal, and fibrin degradation products high.

Which of the following is the most likely diagnosis?

A

Disseminated intravascular coagulation

This is the correct answer. Disseminated intravascular coagulation (DIC) is a syndrome characterised by widespread activation of coagulation pathways, and subsequent depletion of platelets and coagulation factors. Causes include sepsis, haematological malignancies, and major organ dysfunction. Oozing from a cannula size is a classic sign of DIC. The low platelet count and fibrinogen are due to consumption, and prolonged PT due to the consumption of coagulation factors. APTT can be normal or prolonged.

50
Q

The surgical foundation doctor is called to review a 73 year old gentleman on the general surgical ward.

The nursing staff are concerned that he has only passed 200ml of urine in 12 hours and the patient has been nil by mouth overnight, with no IV fluids prescribed.

The patient is taking atenolol, nifedipine, and furosemide.

The patient has dry mucous membranes.

Observations show a heart rate of 97, blood pressure of 101/53, oxygen saturations of 97% on air, respiratory rate of 18, and temperature of 37.1 celsius. Recent blood tests show a creatinine of 140 umol/L (45 - 90 umol/L) and a urea of 20 mmol/L (2.5 - 7.0 mmol/L). W

hich of the following is the single most important management priority in this patient?

A

IV fluid resuscitation

This patient likely has an acute kidney injury secondary to hypovolaemia (i.e. a pre-renal cause), considering he has been nil by mouth, not on any maintenance IV fluids and has a poor urine output.

His regular medications can also be nephrotoxic.

In this case, IV fluid resuscitation is essential to achieve euvolaemia and improve urine output.

If the patient remains nil by mouth he should subsequently be started on maintenance IV fluids.

51
Q

A 25-year-old woman presents to A&E with abdominal pain, drowsiness and vomiting.

She has type 1 diabetes and has recently been less compliant with her insulin regimen.

On examination, the patient has dry mucous membranes, blood pressure 112/78, respiratory rate 24 and pulse rate 110.

Urine dipstick shows ketones +++, blood glucose 25 mmol/L and VBG shows pH of 7.2.

What is the diagnosis?

Which of the following is the next best step in the management of this patient?

A
  • A diagnosis of diabetic ketoacidosis (DKA) is made.
  • 0.9% Sodium Chloride 1L over 1 hour

Although there are some controversial areas in DKA management, it is widely accepted that the most important initial management in DKA is fluid replacement.

Fluid replacement aims to restore circulatory volume, clear ketones, and correct electrolyte imbalance.

A typical adult with DKA has 100ml/kg of water deficit.

If the patient is haemodynamically unstable, e.g. systolic blood pressure < 90, then more aggressive fluid resuscitation would be indicated (e.g. 0.9% sodium chloride 500 mL over 15 minutes).

52
Q

An 18 year old male with no past medical history presents to the emergency department with a decreased conscious level.

Collateral history reveals a few days of vomiting associated with decreased oral intake, and a history of weight loss, increased thirst and increased frequency of urination over the past two weeks.

On examination, the airway is patent and the patient groans in response to painful stimuli, the chest is clear but the respiratory rate is elevated at 32 and oxygen saturations are 96% on room air.

On assessment of circulation, the patient looks pale, with a blood pressure of 89/46 and heart rate of 108bpm.

IV access is available and blood samples, including a blood gas, have been taken.

  • Bedside blood glucose is 26 mmol/L
  • blood ketones are 7.0 mmol/L
  • serum bicarbonate is 13 mmol/L

What is the most likely diagnosis

Which of the following is the single most important inital management consideration?

A

IV fluid resuscitation, followed by maintenance, with 0.9% saline

This patient likely has as-yet undiagnosed type one diabetes and this is their first presentation.

The patient is suffering from diabetic ketoacidosis (DKA), diagnosed by the triad of:

  • hyperglycaemia (>11 mmol/L)
  • ketonaemia (blood ketones >3.0 mmol/L
  • or urine dip >2+ ketones)
  • and acidosis (blood pH <7.30)
  • serum bicarbonate <15 mmol/L

As a result of the uncontrolled hyperglycaemia these patients start to become dehydrated due to the associated polyuria.

As the blood becomes hyperosmolar due to the high glucose levels, more and more water is drawn into the circulation from the intra/extracellular space.

As the condition progresses and ketonaemia/acidaemia develops, patients may start to vomit and oral intake often decreases.

As a result, patients are often profoundly dehydrated on presentation in DKA.

Considering the level of dehydration IV fluid resuscitation followed by maintenance is key, as this will not only support tissue perfusion, but will also help to improve the acidosis and ketonaemia.

There are often associated electrolyte abnormalities in DKA;

  • potassium, chloride, and sodium are often low.
  • As such, giving IV fluid is the most important priority in treating DKA.
53
Q

The patient is suffering from diabetic ketoacidosis (DKA), diagnosed by the triad of: (3)

A

hyperglycaemia (>11 mmol/L)

ketonaemia (blood ketones >3.0 mmol/L

urine dip >2+ ketones)

and acidosis (blood pH <7.30)

serum bicarbonate <15 mmol/L

54
Q

A 74 year old lady presents with

  • severe lethargy, cold intolerance
  • leg swelling
  • decreased consciousness

On examination she is hypothermic, bradycardic and hypotensive.

She has evidence of pulmonary and peripheral oedema and has a round puffy face.

What is the most appropriate initial management of this presentation?

What is she in danger of developing?

A

IV thyroxine

Timely (but careful) administration of T3 or T4 (thyroxine) is critical for reversal of the patient’s symptoms in myxoedema coma.

55
Q

A 75-year old gentleman presents to the A&E department with a 2 hour history of severe crushing central chest pain.

He has a past medical history of HTN, DM and IHD. His basic observations are as follows:

HR 100, RR 25, BP 150/95, T 37.3, SO2 97%

An ECG is performed and is shown below:

What is the most appropriate initial treatment for this patient?

What is the diagnosis?

A

Morphine, Metoclopramide, Aspirin 300mg, Clopidogrel 300mg, GTN spray, PCI

The ECG shows ST elevation in leads aVL, V2 to V5 with reciprocal changes in II, III, aVF.

This is diagnostic of a STEMI. As this patient is saturating at 97%, she does not require oxygen.

The 2008 BTS guidelines recommend that oxygen therapy should only be administered in patients who are saturating less than 94% as there is an increased mortality risk.

These are the most appropriate options for managing a STEMI in the initial setting.

The mnemonic MMONAC can be used to remember this. It stands for Morphine, Metoclopramide, (Oxygen if sats <94%), Nitrates (GTN spray), Aspirin 300 and Clopidogrel 300.

56
Q

The features of life-threatening asthma can be remembered using the mnemonic 33,92 CHEST.

Any one of the following: (7)

A
  • PEF <33%
  • SO2 <92% or PO2 <8
  • Cyanosis
  • Hypotension
  • Exhaustion, altered consciousness
  • Silent chest
  • Tachyarrhythmias
57
Q

A 24-year-old gentleman presents to the Emergency Department with a 1-day history of mild pleuritic chest pain, which he rates at a severity of 1/10.

He is fit and well with no known past medical history.

His basic observations are as follows: HR 88, RR 14, BP 120/80, T 37.0, SO2 99% RA.

On examination, he is comfortable at rest and not in any respiratory distress.

Percussion is hyperresonant over the right lung with a slight decrease in breath sounds.

On palpation, there is no tracheal deviation or crepitus over the chest.

A chest X-ray ordered shows loss of lung markings in upper zone of the right lung, measuring about 1 cm from the chest rim.

What is the best management option for this patient?

A

Discharge, with follow up in 2 to 4 weeks at the outpatient department

The findings on chest examination and signs on the CXR point to a diagnosis of spontaneous pneumothorax.

As this patient does not have any underlying lung disease, this can be considered a primary pneumothorax.

Patients with a primary pneumothorax who are asymptomatic and have a pneumothorax measuring <1cm from the chest rim, as in this case, can be discharged and followed up in 2 to 4 weeks in the outpatient department.

58
Q

A 24-year old male presents to the Emergency Department with nausea, confusion and headaches for the past few weeks. He states his headaches feel like a “tight band” around his head. There is no evidence of associated tonic-clonic movements or trauma. He is normally fit and well.

He is noted to have abnormally pink mucosa on examination. He is afebrile.

What is the most likely diagnosis?

A

Carbon monoxide poisoning

This is a difficult clinical diagnosis to make.

Confusion and pink mucosae are typical features of carbon monoxide poisoning. Most patients also present with a headache, and nausea and vomiting.

In severe toxicity there may be hyperpyrexia, arrhythmias, extrapyramidal features and coma.

Pulse oximetry may be falsely high due to similarities between oxyhaemoglobin and carboxyhaemoglobin.

Treatment is with 100% high-flow oxygen via a non-rebreather mask.

59
Q

A 45-year-old gentleman has a cardiac arrest in hospital.

The resuscitation team is promptly called and begin cardio-pulmonary resuscitation (CPR).

At the first rhythm check, the patient is found to be in ventricular fibrillation (VF) and he is given unsynchronised cardioversion at an energy setting of 200J.

Which of the following is the next best step in management?

A

Continue CPR for 2 minutes

After the first shock, CPR should be commenced immediately for a further 2 minutes after which the rhythm can be checked again.

The pause in chest compressions in order to check the rhythm and administer the shock should be no longer than 5 seconds.

60
Q

A 78-year-old man presents to the Emergency Department with shortness of breath which started two hours ago.

He also complains of chest pain and left calf pain for the last week.

His past medical history includes hypertension, COPD, a stroke three weeks ago and disseminated lung cancer.

His observations include temperature 37.3, blood pressure 100/78mmHg, heart rate 112bpm, respiratory rate 24/minute and Sp02 88% on room air.

An ECG shows sinus tachycardia and a new right bundle branch block.

A CT confirms a massive pulmonary embolism.

What is the most appropriate first-line treatment?

A

Treatment dose of apixaban.

The patient has an acute massive pulmonary embolism (PE) and is in shock.

In this instance thrombolysis would be indicated.

However, the patient has a high risk of haemorrhage due to his recent stroke (an absolute contraindication) and therefore anticoagulation is the next preferred step in management.

Current guidance would suggest a DOAC (such as apixaban) in the first instance.

Being over 75 years of age is also a relative contraindication to thrombolysis.

61
Q

A 67-year-old man on the ward goes into cardiac arrest.

CPR is initiated and airway secured using a laryngeal mask.

Manual defibrillator pads are placed and rhythm analysis shows ventricular fibrillation.

What is the next best step in the management of this patient?

(3)

A

Defibrillation (continuing CPR while defibrillator charges)

This is the correct answer according to the UK Resuscitation Council Advanced Life Support Guidelines (2016).

Ventricular fibrillation and pulseless ventricular tachycardia are shockable rhythms.

Therefore, once identified, one shock should be delivered (at least 150 J of energy).

After this, CPR is resumed for two minutes, and rhythm reassessed.

62
Q

A 60-year-old man presents to the Emergency Department (ED) with severe, sudden onset chest pain.

This was soon associated with weakness in both of his legs.

On examination, the patient is found to have a flaccid paraparesis and a new early diastolic murmur that is loudest over the lower left sternal edge.

What is the most likely diagnosis?

A

Aortic dissection

Aortic dissections commonly present with severe chest pain that is often described as tearing in nature with radiation to the inter-scapular region.

In addition, there are many other possible features that can arise due to blood in the artery wall (false lumen) compressing branches of the aorta.

These include paraparesis (e.g carotid or spinal artery involvement); anuria and loin pain (e.g renal artery involvement); abdominal pain (e.g mesenteric artery involvement) and acute limb ischaemia (e.g subclavian or femoral artery involvement).

Aortic regurgitation is also a recognised complication of aortic dissection and results from a proximal dissection that involves the aortic valve leaflets. This would be the likely cause of the patient’s murmur.

Aortic dissections commonly present with severe chest pain that is often described as tearing in nature with radiation to the inter-scapular region. In addition, there are many other possible features that can arise due to blood in the artery wall (false lumen) compressing branches of the aorta. These include paraparesis (e.g carotid or spinal artery involvement); anuria and loin pain (e.g renal artery involvement); abdominal pain (e.g mesenteric artery involvement) and acute limb ischaemia (e.g subclavian or femoral artery involvement).

Aortic regurgitation is also a recognised complication of aortic dissection and results from a proximal dissection that involves the aortic valve leaflets. This would be the likely cause of the patient’s murmur.

63
Q

An 18 year old man is brought into A&E after taking an overdose of an unknown substance.

He is anxious and complaining of ringing in his ears.

An ABG shows the following:

  • pH7.48
  • PaCO23.4 kPa
  • PaO218 kPa
  • HCO323 mEg/L

What is the likely cause of this overdose?

A

Aspirin

  • Aspirin overdose initially presents with a respiratory alkalosis due to hyperventilation
  • This progresses to a metabolic acidosis.
  • Patients may also experience tinnitus, vomiting and dehydration.
64
Q

A 70-year-old man presents to the emergency department with retrosternal pain.

He describes it as burning and 10/10 in severity.

It came on as he was walking to the shops.

He has had several episodes of this in the last month, but they typically resolved after he stopped and rested for five minutes or so.

In the last week, he has been getting such pains daily.

He has a past medical history of hypertension and hypercholesterolaemia, and is an ex-smoker with a 25 pack-year history.

On examination, he is pale and sweaty, but cardiac, respiratory and abdominal examination are otherwise normal.

Initial ECG is normal. Blood tests have been sent but have not yet returned.

His observations are below:

  • HR 95
  • BP 144/80
  • RR 25
  • SaO2 95%
  • T 36.9

Given the most likely diagnosis, what is the single best management plan?

A

Give aspirin 300mg PO

This patient has acute coronary syndrome.

The history of increasing frequency of chest pain brought on by activity and alleviated by rest is strongly suggestive of crescendo angina.

Without troponin results, it is not yet possible to determine whether this is unstable angina or an NSTEMI.

ECG can initially be normal in both unstable angina and NSTEMI, and serial ECGs should be performed to look for dynamic changes.

65
Q

A 20-year-old gentleman is brought into the A&E Department by his mother.

He has a 2-day of history of headache, nausea and worsening confusion.

His mother noted that he was acting strange today.

He was mumbling to himself and made inappropriate comments about other people.

His is otherwise fit and well with no significant past medical history.

He does not take any regular medication and has no drug allergies.

His basic observations are as follows:

HR 108, RR 20, BP 120/85, T 38.2, SO2 97%

On examination, he appears confused with an AMTS of 8. Auscultation of his heart and lungs are clear.

His neck movements appear stiff and you note a blanching erythematous maculopapular rash across both his legs. While assessing his neurology, he had difficulty following instructions but he appeared to have some weakness in his lower limbs.

Which of the following is the most appropriate treatment for this gentleman?

A

IV Ceftriaxone and IV Acyclovir

In addition to fever, headache and vomiting, this patient also shows features of altered mental status, personality change and possibly focal neurological deficits. Encephalitis must be suspected and empirical treatment should include an anti-bacterial and anti-viral agent, therefore this is the right answer.

66
Q

A 25 year old male is admitted following a road traffic accident.

On assessment his;

  • heart rate is 128bpm,
  • respiratory rate 38/min,
  • blood pressure 86/52,
  • oxygen saturation 98% on room air

He has an obvious deformity of his pelvis but no external or penetrating injuries.

What is the most appropriate initial management of this patient?

(3)

A

Give crystalloid IV fluid and assess response

This is the recommended course of action according to the Advanced Trauma Life Support algorithm warmed crystalloids should be used for initial resuscitation.

Further decisions about whether the patient requires blood are then made based on the patient’s response to initial fluid.

67
Q

A 28 year old man with Type 1 diabetes presents complaining of tiredness, nausea, polyuria and polydipsia.

On examination he had dry mucous membrane, a non-visible JVP and tachycardia.

He is normotensive.

  • glucose returns at 19 mmol/L
  • ketones come back at 3.4 mmol/L
  • pH 7.28
  • bicarbonate 16.2 mmol/L
  • sodium 151 mmol/L
  • potassium 4.3 mmol/L.

What is the most appropriate immediate management of this patient?

A

1L IV normal saline over 1 hour

The most important part of diabetic ketoacidosis is not the blood sugar but instead the dehydration (as this is what is most life-threatening).

It is therefore important to give IV fluids quickly to restored body fluid volume.

The patient’s hypernatraemia is a consequence of his dehydration and is only a relative hypernatraemia (i.e. more water has been lost than sodium) therefore there is no problem giving normal saline in this situation.

68
Q

An 81 year old man had a myocardial infarction 3 days ago, which was found to be due to occlusion of the left anterior descending artery.

He has now started to score highly on the NEWS chart with low blood pressure and high heart rate.

On examination he had a raised JVP.

On auscultation there is a murmur which is harsh and holosystolic.

What is the most likely complication?

A

Rupture of the interventricular septum

Patients with occlusion of the left anterior descending artery are particularly at risk of rupture of the interventricular septum due to infarction of the septal area.

The rupture usually leads to haemodynamic instability, with hypotension and biventricular failure (often largely right-sided).

An auscultation a harsh and holosystolic murmur may be heard.

69
Q

A 20-year-old male is brought in by ambulance to the Emergency Department.

He was a restrained front-seat passenger in a road traffic accident.

He is complaining of left sided pleuritic chest pain and shortness of breath.

His vital signs are stable and he is given analgesia and sent for a chest x-ray.

While he is in the x-ray department he becomes more short of breath.

His heart rate begins to rise and his blood pressure begins to fall.

You are called to review him in the x-ray room as he is quickly becoming unstable.

You review the x-ray that has just been taken.

What is the next immediate step that should be taken?

What exact treatment is needed?

A

Urgent needle decompression followed by chest drain insertion

This patient has a left sided apical pneumothorax, but worryingly the mediastinum is shifted towards the right.

This is highly concerning for a tension pneumothorax and needs to be urgently decompressed using a wide bore needle/cannula in the 2nd intercostal space, midclavicular line and followed up with a chest drain.

70
Q

A 72-year old man was admitted to the hospital following a fractured neck of femur.

He suddenly becomes unwell 5 days after his operation.

He is complaining of chest pain and shortness of breath.

The pain is worst on deep inspiration.

He has a past medical history of COPD, hypertension and hypercholesterolaemia.

There are no clinical findings on examination.

His observations include

  • temperature 37.8
  • blood pressure 102/84mmHg
  • heart rate 110bpm
  • respiratory rate 24/minute
  • Sp02 86% on room air

What is the most likely diagnosis?

A

Pulmonary embolism (PE)

The patient is at higher risk of developing a PE due to the immobility following his operation.

He is describing pleuritic chest pain, is tachycardic and tachypnoeic. Patients may also present with cough and haemoptysis.

In severe cases, right heart failure causes dizziness or syncope.

71
Q

A 61 year old male presents to the emergency department with severe back pain. He has a history of prostate cancer with metastases to the spine.

His neurological examination reveals decreased power and sensation in both lower limbs, and he complains of new onset bladder incontinence.

What is the most appropriate treatment for this patient?

A

Dexamethasone

This patient presents with symptoms of spinal cord compression, and needs urgent neurosurgical review.

A loading dose of dexamethasone should be given whilst awaiting specialist advice.

72
Q

A 15 year old female patient presents to the Emergency Department with difficulty in breathing following eating a slice of cake at a birthday party.

She is known to have an anaphylactic reaction to peanuts and it is thought that the cake was contaminated with the allergen.

Which added sound is most likely to be heard from the end of the bed when listening to this patient breathing?

A

Stridor is the term used to describe an inspiratory wheeze which occurs when the airway swells, such as in anaphylaxis or in infections such a croup (a viral infection common in young children) or epiglottitis (a bacterial infection which is rarely seen since the introduction of the H. influenzae vaccine into the childhood vaccination regime).

73
Q

A 82-year-old gentleman, with a past medical history of end-stage chronic kidney disease and congestive heart failure, presents with difficulty in breathing, especially on lying down, and pink frothy sputum.

On examination, he appears anxious, his JVP is raised and bi-basal crackles are auscultated.

His observations are as follows: HR 72, BP 88/58, SpO2 93% RA, RR 26, Temp 36.9 degrees Celsius.

Severe pulmonary oedema is diagnosed, for which morphine, metoclopramide and furosemide have been administered.

The attending physician is now considering the use of a glyceryl trinitrate (GTN) infusion.

Which single piece of information above would contraindicate the use of a GTN infusion in this patient?

A

BP 88/58

GTN is notoriously known to cause a decrease in the blood pressure, and is contraindicated in patients whose systolic pressure is less than 90 mmHg. This could potentially cause the patient to have a cardiac arrest.

Note that the systolic pressure cut-off should be used in the clinical context appropriately - there is no guarantee the blood pressure will not drop precipitously if it were, for example, 91 mmHg to start with. Other treatments for pulmonary oedema such as morphine and furosemide all cause a fall in the blood pressure, hence it is an important parameter to observe in this context.

74
Q

A 73 year old gentleman presents to the emergency department with his wife following an episode of collapse.

The patient has atrial fibrillation, and takes regular apixaban, bisoprolol, and amlodipine.

The patient describes getting out of bed in the morning, having had a poor night’s sleep and feeling dizzy briefly before everything went dark.

He then remembers being woken by his wife a few seconds later.

The patient’s wife describes that the patient went pale and slumped to the floor, with brief jerking of his arms and legs.

After the patient woke up he was fully alert and conscious within a few minutes.

On examination the patient is alert and orientated, respiratory examination is unremarkable, cardiovascular examination and 12-lead ECG confirms the patient is in atrial fibrillation.

There is no focal neurological deficit, and observations are all within normal parameters.

Which of the following is the single most likely diagnosis?

A

Orthostatic syncope

Also known as orthostatic hypotension or postural hypotension, this is a diagnosis that is more common in elderly patients and/or those who are taking medications that can cause low blood pressure.

This patient is both elderly (so likely has a degree of autonomic dysfunction) and takes medications that may lower his blood pressure (bisoprolol and amlodipine).

Factors that support this diagnosis are that the patient had been lying in bed overnight and rapidly stood up in the morning, the episode was preceded by a brief period of dizziness, and the patient rapidly regained consciousness and was behaving normally within a few minutes after the episode.

75
Q

A 19-year-old lady presents to the Emergency Department (ED) having taken an overdose of an unknown substance.

She reports feeling nauseous and dizzy with an irritating ringing sound in both ears.

Her respiratory rate is 30 breaths per minute and her pulse rate is 90 bpm. She is haemodynamically stable.

An arterial blood gas (ABG) reveals:

  • pH: 7.50 (7.35-7.45)
  • PaCO2: 4.0 kPa (4.5-6.0 kPa)
  • PaO2: 14.5 kPa (11-15 kPa)
  • Bicarbonate: 24 mmol/l (22-28 mmol/L)

What is the most likely substance that the patient has taken an overdose of?

A

Aspirin

Tinnitus is a characteristic feature of an aspirin overdose.

Other symptoms include vertigo, nausea, sweating, hyperventilation and confusion.

The ABG shows a respiratory alkalosis that is secondary to hyperventilation induced by the drug stimulating respiratory centres in the brain-stem.

A follow-up ABG at a later point may show a raised anion-gap metabolic acidosis.

This is due to aspirin being a salicylate and therefore intrinsically acidic.

Management consists of measuring serum salicylate levels and acting accordingly.

Options include increasing fluid intake, alkalinising the urine (to facilitate excretion of the drug) and dialysis.

76
Q

A 65-year old woman presents to the A&E Department with a 2 hour history of chest discomfort and difficult breathing.

She has a past medical history of poorly controlled SLE.

Her basic observations are as follows:

HR 110, RR 25, BP 80/60, SO2 92%

On examination, she appears very anxious and has some difficulty breathing.

Her JVP is raised and there is some pedal oedema.

On auscultation, heart sounds are difficult to hear.

You notice that when she breathes in, her JVP rises and her systolic blood pressure drops.

Given the underlying diagnosis, what is the most appropriate management?

A

Insert a needle just left to her xiphoid process, aiming towards her left shoulder

This describes how pericardiocentesis can be done. Pericardiocentesis involves the insertion of a needle into the pericardial sac to relieve over-accumulation of fluid.

This can be done to treat cardiac tamponade, which is what this lady has. SLE is a risk factor for the development of cardiac tamponade. This lady also displays Beck’s triad - the combination of raised JVP, hypotension and muffled heart sounds, a feature of cardiac tamponade.

She has Kussmaul’s sign (rise in JVP with inspiration) and pulsus paradoxus (drop in systolic blood pressure of about 15 mmHg with inspiration), which are also features of cardiac tamponade.

77
Q

A 29-year old woman is admitted to the Emergency department. She states she has had on-going headaches for which she has been taking over the counter medications. She is feeling short of breath and dizzy. She denies any pain and she is normally fit and well.

Her observations are stable.

An arterial blood gas (ABG) is done which shows a respiratory alkalosis.

Which drug is the most likely cause of the patient’s symptoms?

A

Aspirin

The arterial blood gas (ABG) shows a respiratory alkalosis.

In salicylate poisoning, for example in aspirin overdose, respiratory alkalosis occurs early due to stimulation of the brainstem medullary respiration centre.

This is later followed by metabolic acidosis. This early respiratory alkalosis may present with shortness of breath alone.

Aspirin is the most common salicylate in regular use and poisoning is common.

Management includes possible gastric lavage, activated charcoal, hydration and consideration for dialysis in severe cases.

78
Q

A 52-year-old woman presents to A&E with a one-day history of abdominal pain and decreased appetite.

On examination, the abdomen is soft and tender maximally in the right upper quadrant.

Bowel sounds are present.

Yellow discolouration of her skin and conjunctiva is noted.

Latest observations show respiratory rate 12/min, SpO2 96% on room air, pulse rate 110/min, blood pressure 120/78, temperature 38.7.

Which of the following is the most likely diagnosis?

A

Ascending cholangitis

Charcot’s triad of ascending cholangitis:

  • right upper quadrant (RUQ) pain
  • fever/raised white cell count
  • jaundice

This is an infection of the bile duct which must be treated with systemic antibiotics.

This is NOT Acute cholecystitis

Right upper quadrant pain and fever are features of acute cholecystitis; however, jaundice indicates obstruction at the level of the common bile duct leading to cholangitis.

79
Q

What is Reynolds’ pentad?

A

Reynolds’ pentad describes two further features in addition to Charcot’s triad:

  • shock (low blood pressure)
  • altered mental state
  • RUQ pain
  • Jaundice
  • fever/raised white cell count

Used to diagnosis ascending cholangitis

80
Q

A 23 year old man develops acute angioedema and stridor after consuming some food from a market.

On initial assessment, he is in respiratory distress and covered in an urticarial rash.

Which of the following is the single most appropriate initial treatment option?

A

0.5ml 1 in 1000 adrenaline IM

Angioedema, urticaria, stridor and respiratory distress are important symptoms of anaphylaxis. In this case, the trigger is likely to be a substance in food that this man has consumed.

1 in 1000 adrenaline means 1g (or 1000mg) in 1000mls. This simplifies to 1mg per 1ml of solution. Therefore 0.5mls of 1 in 1000 adrenaline gives a dose of 500 micrograms (0.5mg) of adrenaline. This is the recommended dose for anaphylaxis in patients aged over 12. Further doses can be given at 5-minute intervals if necessary.

The preferred route is IM as treatment can be started without gaining IV access. IV administration is also extremely dangerous and should not be performed by anyone other than a trained specialist (e.g. anaesthetist).

81
Q

You are called to see a 70-year-old inpatient who has developed a fever and abdominal pain.

He is five days post elective laparoscopic anterior resection with end anastamosis for bowel cancer.

On examination, he has a rigid abdomen with generalised tenderness, and no obvious surgical wound infection.

There is purulent discharge from the abdominal drain.

Auscultation reveals decreased bowel sounds.

Heart sounds and breath sounds are normal.

His observations are below:

  • HR 130
  • RR 28
  • BP 93/59
  • SaO2 94%
  • T 38.2

Given the most likely diagnosis, what is the single best diagnostic imaging?

A

CT abdomen and pelvis with rectal contrast

This patient has developed a colonic anastamotic leak, which is one of the most serious complications of intestinal surgery.

This is indicated by his fever, tachycardia, rigid abdomen, and purulent abdominal drain.

A CT abdomen and pelvis with rectal contrast can the detect presence and location and degree of a leak which will guide further management.

A complimentary or alternative test is a fluoroscopic water-soluble enema.

82
Q

An 80-year-old man is admitted to the emergency department with abdominal pain and vomiting. He denies diarrhoea and haematemesis.

He has a background of severe COPD and has had four admissions in the last two months for non-infective exacerbation of COPD, for which he received antibiotics and oral steroids per admission.

He admits to poor compliance with medications.

On examination, he has dry mucous membranes and loss of skin turgor. Respiratory and abdominal examination are unremarkable.

Initial investigations reveal

  • Na+ 130 mmol/L
  • K+ 5.2 mmol/L
  • urea 8.2 mmol/L
  • glucose 2.9 mmol/L

His observations are below:

  • HR 120
  • BP 70/45 (standing) 90/52 (lying)
  • RR 17
  • T 37.3
  • SaO2 90%

Given the most likely diagnosis, what is the single best definitive treatment?

A

Hydrocortisone 100mg IM

This patient has an iatrogenic Addisonian crisis due to repeated prescriptions of steroids with poor compliance to medications, and no consideration by his prescribers for the effect of previous prescriptions.

It is an example of a medical cause of the ‘acute abdomen’.

Patients with an Addisonian crisis can present variably such as with shock, abdominal pain or hypoglycaemia.

It can be accompanied by mild hyponatraemia and hyperkalaemia.

83
Q

A pedestrian has a witnessed fall down a flight of stairs.

He is found unresponsive with no signs of life.

An ambulance is called for immediately and basic life support is initiated.

Due to the dangerous mechanism, a cervical spine injury is suspected.

Which of the following is the best way to manage the airway in this setting?

A

Jaw thrust

According to the UK Resuscitation Council, in a patient with a suspected cervical spine injury, there is a theoretical risk of further damage to the cervical-spine when performing head tilt.

Therefore, a jaw thrust should be performed alone.

It should be noted, that if there is life-threatening airway obstruction, head tilt should be performed incrementally until the airway opens (patent airway takes priority over potential cervical spine injury).

84
Q

A 24-year-old woman presents to A&E with palpitations that began two hours ago.

She denies chest pain, shortness of breath or dizziness.

Observations are as follows: respiratory rate 10/min, SpO2 96% on room air, pulse rate 125/min, blood pressure 124/88 and temperature 37.2.

A 12-lead ECG is performed which shows a regular narrow complex tachycardia.

An atrioventricular nodal re-entry tachycardia (AVNRT) is suspected.

Which of the following is the next best step in the management of this patient?

A

Vagal maneuvers

This patient has a narrow complex tachycardia with no adverse features (myocardial ischaemia, syncope, shock, heart failure).

Therefore, the most appropriate first step is to attempt vagal maneuvers such as carotid massage or valsalva maneuver (asking the patient to blow into a syringe) which is an attempt to transiently block the atrioventricular node.

If this is unsuccessful, IV Adenosine can be given.

85
Q

A 21 year old type 1 diabetic presents with a 3 day history of tiredness, polyuria, polydipsia and nausea.

The patient is clinically dry but examination is otherwise unremarkable.

A capillary blood glucose returns at 19.5mmol/L. A VBG shows a pH of 7.27 and bicarbonate of 13.2mmol/L.

An ECG shows small T waves and first degree heart block.

What complication does the patient most likely have?

A

Hypokalaemia

Potassium disturbance is the most common complication of DKA.

The presence of small T waves and prolonged PR interval shows that the patient has hypokalaemia.

86
Q

An 18-year-old woman with known epilepsy presents to the emergency department with ongoing seizures.

She has been seizing for 20 minutes.

Paramedics have already administered oxygen and one dose of Diazepam 10mg PR 15 minutes ago enroute.

IV access has not been established.

What is the single best next treatment?

A

Repeat rectal Diazepam 10mg PR

Rectal Diazepam can be repeated once after 10-15 minutes if the first dose is ineffective and IV access has not been established.

87
Q

A 64 year old Type 2 diabetic presents with a week long history of tiredness, polyuria, polydipsia and weakness.

He appears clinically dehydrated but his observations are stable.

A capillary blood glucose returns at 32mmol/L. Capillary ketones are 0.1mmol/L.

Blood tests are done which return as follows:

  • Na 150 (135-145)
  • K 5.5 (3.5-5.5)
  • Ur 9.3 (2.5-6.7)
  • Cr 193 (70-150)

What is the most appropriate initial step in the management of this patient?

A

1L 0.9% saline over 1-2 hours

The most important initial step in HHS (Hyperosmolar Hyperglycaemic State) management is reducing serum osmolality by treating the dehydration.

The best way to do this is to start by giving normal saline over 1-2 hours.

Note that initial resuscitation is slightly slower than in DKA (Diabetic Ketoacidosis).

This because the dehydration in HHS develops over a long time period and is more pronounced so should be corrected slightly more cautiously to avoid rapid fluid shifts.

88
Q

A 45-year-old medical patient reports sudden onset shortness of breath and chest pain.

Following initial assessment, the patient is found to have a Wells’ score of 6 and is sent for a CT pulmonary angiogram.

This reveals a saddle pulmonary embolism (PE).

On his return to the ward, the patient is found to be hypoxic with a systolic blood pressure of 75 mmHg.

Which of the following is the most appropriate next step in definitive management?

A

IV thrombolysis

This patient has a ‘massive (high-risk) pulmonary embolus’ as he has evidence of haemodynamic instability (systolic blood pressure < 90 mmHg or a drop of 40 mmHg for 15 minutes or more).

The recommended management for a massive PE is thrombolysis, however it should be noted that it carries a significant risk of intracerebral bleeding.

89
Q

A 70-year-old gentleman presents to the Emergency Department with worsening dyspnoea and productive cough over the last two days.

He has a past medical history of COPD and has a 40 pack year smoking history.

His regular medications include PRN salbutamol and a regular seretide inhaler.

His observations are as follows: HR 100, RR 24, BP 140/90, T 37.9, SO2 82% RA.

He is given 60% oxygen via a green Venturi mask.

He was also started on ipratropium nebulisers, prednisolone and amoxicillin.

About half an hour later, you come back to reassess the patient and the nurse states that he has become drowsy and slightly confused.

He also complains of a slight headache.

On auscultation, you note there is less wheeze than before.

You note that his O2 saturation readings have improved to 98%.

What is the most appropriate next management step?

A

Reduce oxygen supply to 20%

This patient has started to become drowsier, confused and developed a headache about half an hour after being started on 60% oxygen.

In the context of a COPD patient, this must be treated as hypercapnia. The oxygen supply should be reduced immediately.

You may also need to do an ABG to assess his arterial pH and pCO2, which will allow you to consider further management options such as NIV.

90
Q

A 48 year old female has presented to the emergency department with a fever and feeling generally unwell.

Initial observations are:

  • heart rate 109,
  • respiratory rate 24,
  • oxygen saturations 94% on air,
  • blood pressure 94/67,
  • temperature 38.7 Celsius.

The patient is alert and responsive, but confused.

What is the next most appropriate action?

  • Take a blood gas sample
  • Give intravenous broad-spectrum antibiotics
  • Apply 15L/min oxygen via non-rebreathe facemask
  • Give oral paracetamol 1g
  • Give an IV fluid bolus of 500ml 0.9% saline
A

Apply 15L/min oxygen via non-rebreathe facemask

The history is consistent with sepsis.

The patient is alert and responsive, so there is no suggestion that the airway is compromised.

‘Critically ill’ patients, including those with sepsis, should receive high-flow oxygen irrespective of their saturations, as per British Thoracic Society guidelines.

This is on the basis that there is likely a degree of underlying tissue hypoperfusion and associated hypoxia, even in the presence of normal saturations.

91
Q

A 34 year old female presents to the emergency department acutely short of breath.

Her respiratory rate is 34, oxygen saturations 87% on 15L/min O2, heart rate 127 and blood pressure 83/42.

The patient is still talking but with some difficulty.

There is a diffuse maculo-papular rash across her torso and she had been complaining of associated itching too.

What single best immediate action?

A

Prepare and administer 500 micrograms of IM adrenaline (1:1000 concentration)

This patient is showing signs of anaphylactic shock, this is the correct dose/route/concentration of adrenaline to administer in anaphylaxis.

This treatment will both help to reduce the airway angioedema, which may cause rapid airway compromise, as well as supporting the circulation by increasing the blood pressure (by increasing peripheral vascular resistance and its intotropic/chronotropic effects).

92
Q

A 67-year-old man with metastatic prostate cancer presents to A&E with worsening back pain which radiates down his legs.

He also notes worsening urinary incontinence over the last few days.

A full neurological exam is performed which shows decreased sensation and power in the legs bilaterally.

A diagnosis of spinal cord compression is suspected.

Which is the following is the next best step in the management of this patient?

A

Whole spine MRI

This is the correct answer. Spinal cord compression is a medical emergency and once suspected, must be investigated radiologically. MRI whole spine is the modality of choice which will confirm the diagnosis and assess the likely cause of the compression.

93
Q

An elderly gentleman had been brought in by ambulance (BIBA) on a cold January morning. He was found by his morning carer, slumped in an armchair. The carer was concerned that he was more confused and less responsive than normal. He has vomited and is unable to mobilise independently (he is normally mobile). Upon arrival to the emergency department, he appears flushed and his vital signs are as follows:

  • Respiratory rate = 21
  • Heart rate = 109
  • Blood pressure = 110/74
  • Temperature = 37.1 degrees celcius
  • Oxygen saturation = 100%

Which investigation can be performed initially in the emergency department to support the diagnosis?

A

ABG with co-oximetry

The most likely diagnosis here is carbon monoxide poisoning. Carbon monoxide can be produced by faulty boilers and results in confusion and drowsiness. Pulse oximetry gives a falsely high reading as is it unable to distinguish between carboxyhaemoglobin and oxyhaemoglobin. An ABG should be performed and analysed using a co-oximeter to differentiate between carboxyhaemoglobin and oxyhaemoglobin.

94
Q

A 78 year old man sitting in the discharge lounge awaiting transport begins to experience chest tightness and sweating.

A nurse takes bloods and performs an ECG.

A doctor arrives later.

The pain is still present but has subsided slightly.

The FBC, U&Es and troponin are all normal.

What is the most appropriate next step?

A

Repeat the troponin and review the patient again

As an elderly male this patient has an increased risk of a cardiovascular event. The chest tightness and sweating suggest this could be an ischaemic event. Troponin does not rise until 2 to 3 hours after myocardial infarction and so the troponin needs to be repeated to identify any changes.

95
Q

A 35-year-old man presents to the Emergency Department with a history of increasing confusion over the past 2 hours.

He also complains of a bad headache and is feeling nauseous.

He had just returned from a ski holiday yesterday.

While skiing, his family recalls an incident where he fell over and hit his head against some rocks.

However, he appeared fine and walked away from that accident without any problems.

Currently, his basic observations are as follows:

HR 67, RR 30, T 36.7, SO2 94% RA, BP 150/95

What is the most likely cause of his current presentation?

A

Damage to middle meningeal vessels

Damage to the middle meningeal vessels would result in an Extradural haemorrhage (EDH).

Patients with an EDH usually experience a lucid interval after a recent traumatic incident.

This refers to a period where there is temporary improvement in a patient’s condition following injury, after which the condition deteriorates.

This patient has a history of a lucid interval following his ski accident.

A few days later, he has started to deteriorate, showing signs of raised intracranial pressure (ICP), probably resulting from an extension and worsening of the bleed in his brain.

96
Q

55-year old man presents to the A&E department with a 1 day history of palpitations.

He has a past medical history of childhood asthma and schizophrenia. He was started on a new antipsychotic yesterday. His basic observations are as follows:

HR 100, RR 25, BP 125/80, T 37.3, SO2 96%

On examination, JVP is not visible and there was no peripheral oedema. Auscultation of his heart and lungs were normal.

A portion of his ECG is shown below:

What is the most appropriate management for this patient?

A

IV Magnesium Sulphate

IV Magnesium Sulphate is the most appropriate treatment for TDPs, which is what this patient has on ECG. Anti-psychotics can cause a prolonged QT interval, which can develop into TDP.

97
Q

A 25-year-old man presents to the emergency department with vomiting and drowsiness.

He has a background of Type I Diabetes mellitus and uses a basal-bolus regimen.

There is a two-day history of abdominal pain and diarrhoea and, as a result of poor appetite, he has stopped using the short acting Insulin.

His urinalysis results are below:

  • Urine dip
  • Leukocyte esterase nil
  • Nitrite nil
  • pH 5.6
  • Protein nil
  • Blood nil
  • Ketones +++
  • Glucose +++
  • Bilirubin nil

Given the most likely diagnosis, what is the single best initial treatment?

A

Start IV crystalloids

Fluid replacement is the most important initial treatment in DKA as there can be severe fluid deficits.

Fluid replacement will also help lower blood sugar.

98
Q

A 65-year-old gentleman presents to the Emergency Department.

He had an episode of severe crushing central chest pain which radiated to his left arm 1 day ago.

He also felt nauseous and sweaty during that episode.

He did not come to the Emergency Department then as he thought it was nothing serious and the pain had since resolved.

He has a past medical history of HTN, DM, IHD and stroke, which he suffered 2 months ago.

His regular medications include amlodipine, metformin, enalapril, atorvastatin and warfarin.

His basic observations are as follows: HR 90, RR 20, BP 155/95, T 37.3, SO2 97% RA.

He is currently asymptomatic and cardiac and respiratory examinations are entirely normal.

He is immediately given aspirin and clopidogrel.

An ECG is performed and it is shows ST elevation in V1 to V3 with reciprocal changes.

His serum troponin is also raised.

Unfortunately, the hospital is very busy today and after 2.5 hours of waiting, he had not been seen again and had not received any further treatment.

Given the underlying diagnosis, what is the most appropriate treatment for this patient now?

A

Pharmacological management only

This patient has clinical features and evidence of a STEMI.

ST elevation can last many hours after the episode, and troponin levels reach a peak at about 12 to 24 hours after the episode of chest pain. However, he does not meet the time window for PCI.

PCI should only be offered within 12 hours of symptom onset.

Beyond 12 hours, PCI can still be offered if the patient has persistent symptoms.

In this case, the patient is currently well and asymptomatic.

He has certain contraindications to thrombolysis which include recent stroke (within 3 months) and being currently on anticoagulation.

This patient should only receive pharmacotherapy for his ACS.

This usually includes aspirin, Ticagrelor or Clopidogrel, Fondaparinux in the acute stages followed by bisoprolol and a statin.

99
Q

A 60-year-old man is brought to the emergency department complaining of ‘butterflies’ in the chest.

He has had the chest symptoms for the past week, but woke up this morning feeling additionally dizzy, unwell and short of breath.

He admits to heavy alcohol consumption of around 30 units a week.

On examination, he is pale and sweaty with cool peripheries.

He has an irregularly irregular radial pulse and capillary refill time of 4 seconds.

On auscultation, there are bibasal crackles on inspiration.

ECG shows an absence of P waves, with irregularly irrregular narrow QRS complexes.

IV fluids and oxygen are being administered.

His observations are below:

  • HR 230
  • BP 80/44
  • RR 28
  • SaO2 93%
  • T 37.1

Given the likely diagnosis, what is the single next best step?

A

Fast Atrial Fibrillation

Synchronised DC cardioversion

This patient is showing signs of haemodynamic instability:

  • tachycardia > 200
  • heart failure (pulmonary oedema)
  • shock

The treatment for any tachycardia with adverse features is DC cardioversion, after ABCDE measures have been taken.

100
Q

A 25-year-old lady comes into the Emergency Department having ingested 30 tablets of paracetamol about 3 hours ago. She is currently asymptomatic. Her observations are as follows: HR 100, RR 18, BP 130/80, T 37.0, SO2 99% RA.

According to local protocol, she would have to wait an hour and have her paracetamol levels tested at 4 hours before starting treatment. However, the Emergency Department consultant wants to start her on NAC treatment now.

Which of the following factors would likely be the reason for the consultant’s decision?

A

History of anorexia nervosa

There is increased risk of paracetamol toxicity in patients that are in glutathione deplete states. This includes eating disorders, HIV and malnutrition. A history of anorexia nervosa would warrant immediate administration of NAC.

101
Q

A 22-year-old builder is brought to the emergency department by ambulance after sustaining a head injury at work.

The patient was described to be initially unconscious at the building site but is now awake and complains of severe pain around his head.

On examination, GCS is 13 with no signs of an open skull fracture.

Since arriving at the emergency department the patient has vomited twice.

What is the correct next step?

A

CT head <1 hour

Given the history of initial unconsciousness following the head injury, followed by a lucid interval, a diagnosis of an extradural bleed should be considered.

This patient has vomited twice since his head injury and has failed to return to GCS 15. NICE recommends a CT head within one hour.

Even without knowledge of the specific NICE criteria, this clinical picture should be recognised as an emergency.

102
Q

A 70-year-old man is brought to the emergency department by ambulance from home.

His daughter is concerned that he appears to be becoming more and more confused and drowsy.

His past medical history includes Type 2 diabetes, Hypertension and Hypercholesteremia.

On examination the patient is very confused with dry mucous membranes and reduced skin turgor.

Capillary refill time is over three seconds.

Cardiovascular and respiratory exam are otherwise normal.

Observations

  • HR 110
  • BP 90/60
  • RR 18
  • SaO2 98% on air
  • T 370C
  • Blood glucose is 37
  • Urine dip is negative

Blood results

  • Hb 135
  • Wcc 12.3
  • Na 138
  • K 5.6
  • Cr 86

Give the likely diagnosis

What is most appropriate initial treatment to start?

A

0.9% saline IVI

This patient is severely dehydrated due to hyperosmolar hyperglycaemic state (HHS), as evidenced by:

  • hypotension
  • tachycardia
  • confusion

The history of Type 2 diabetes, severe hyperglycaemia, and the lack of other significant examination findings points to the aetiology.

He requires resuscitation with IV fluids.

Even without knowing the aetiology, this patients’ need for fluids and risk of VTE should be recognised.

103
Q

A 65-year old gentleman presents to the A&E department.

He had an episode of severe crushing central chest pain which radiated to his left arm 1 day ago.

He also felt nauseous and sweaty during that episode.

He did not come to A&E then as he thought it was nothing serious and the pain had since resolved.

He has a past medical history of HTN, DM, IHD and stroke, which he suffered 2 months ago. His regular medications include amlodipine, metformin, enalapril, atorvastatin and warfarin.

His basic observations are as follows:

HR 90, RR 20, BP 155/95, T 37.3, SO2 97%

He is currently asymptomatic and cardiac and respiratory examinations are entirely normal.

He is immediately given aspirin and clopidogrel.

An ECG is performed and it is shows ST elevation in V1 to V3 with reciprocal changes.

His serum troponin is also raised.

Unfortunately, the hospital is very busy today and after 2.5 hours of waiting, he had not been seen again and had not received any further treatment.

Given the underlying diagnosis, what is the most appropriate treatment for this patient now? (3)

A

Pharmacological management only

This patient has clinical features and evidence of a STEMI. ST elevation can last many hours after the episode, and troponin levels reach a peak at about 12 to 24 hours after the episode of chest pain.

However, he does not meet the time window for PCI.

PCI should only be offered within 12 hours of symptom onset.

Beyond 12 hours, PCI can still be offered if the patient has persistent symptoms. In this case, the patient is currently well and asymptomatic.

He has certain contraindications to thrombolysis which include recent stroke (within 3 months) and being currently on anticoagulation.

This patient should only receive pharmacotherapy for his ACS, including anti-platelet therapy and beta-blockers

104
Q

A 33 year old gentleman with no significant past medical history is involved in a road traffic accident and was rushed into A&E.

His basic observations are as follows:

HR 110, RR 35, BP 70/50, SO2 88%

He is in severe respiratory distress.

After a quick initial assessment, the consultant decides to perform an immediate needle decompression with a large-bore needle inserted into his right 2nd intercostal space midclavicular line.

Given the underlying diagnosis, which of the following examination findings would the consultant most likely have elicited?

A
  • Tracheal deviation to the left
  • reduced chest expansion
  • hyper-resonant percussion on the right
  • decreased vocal resonance on the right

This option describes the typical examination findings in a patient with a tension pneumothorax.

A tension pneumothorax may be large enough to shift the trachea to the opposite side.

Due to the collection of air in the pleural space, percussion will appear hyperresonant and vocal resonance will be decreased on the same side as the pneumothorax.

Other findings may include signs of haemodynamic instability and crepitus over the skin from surgical emphysema.

105
Q

A 30-year-old woman is brought into the emergency department via ambulance, following a domestic altercation involving physical violence. She complains of chest pain and breathlessness. On examination there is noticeable bruising on her face, arms and bilaterally on her thorax. Her trachea is deviated to the left. Chest expansion is reduced on the right side. Breath sounds are absent on the right side. There is increased resonance to percussion on the right side. She is increasingly drowsy.

Observations

  • HR 150
  • BP 70/45
  • RR 30
  • SaO2 89%
  • T 37.1

What is the single best next step? (4)

A
  • Insert a 16G cannula
  • into 2nd intercostal space
  • mid-clavicular line
  • right side

This patient has a right-sided tension pneumothorax secondary to blunt trauma.

A 14-16G cannula (i.e. orange or grey) should be inserted on the ipsilateral side without delay to relieve the tension.

106
Q

A 28 year old known type 1 diabetic is admitted with a 4 day history of polyuria, polydipsia, tiredness, nausea and vomiting.

A capillary blood glucose returns at 18.2mmol/L and capillary ketones are 3.6mmol/L.

What is the most appropriate investigation to confirm the most likely diagnosis? (3)

A

Venous blood gas

  • pH and the bicarbonate levels,
  • either of which if low will confirm a diagnosis of diabetic ketoacidosis.
107
Q

A 24-year-old gentleman presents to the A&E Department with a 1 day history of mild pleuritic chest pain, which he rates at a severity of 1/10.

He is fit and well with no known past medical history.

His basic observations are as follows:

HR 88, RR 14, BP 120/80, T 37.0, SO2 99%

On examination, he is comfortable at rest and not in any respiratory distress.

Percussion is hyper-resonant over the right lung with a slight decrease in breath sounds.

On palpation, there is no tracheal deviation or crepitus over the chest.

A Chest X-ray ordered shows loss of lung markings in the upper zone of the right lung, measuring about 1cm from the chest rim.

What is the best management option for this patient?

A

Discharge, with follow up in 2 to 4 weeks at the outpatient department

The findings on chest examination and signs on the CXR point to a diagnosis of spontaneous pneumothorax.

As this patient does not have any underlying lung disease, this can be considered a primary pneumothorax.

Patients with a primary pneumothorax who are asymptomatic and have a pneumothorax measuring <1cm from the chest rim, as in this case, can be discharged and followed up in 2 to 4 weeks in the outpatient department.

108
Q

A 52-year-old man on the Gastroenterology ward has just had an episode of large volume haematemesis.

On examination he is cool and clammy to touch, pulse rate is 130 bpm and blood pressure is 87/42.

An upper gastrointestinal bleed is suspected.

Two large-bore cannulae have been inserted.

Which of the following is the next best management of this patient?

A

Rapid infusion of IV 0.9% Sodium Chloride 1 L

Specific protocol for acute upper gastrointestinal bleed may differ between hospitals; however, this patient is haemodynamically unstable and so must be resuscitated with rapid infusion of crystalloid.

109
Q

A 68-year-old woman presents to A&E with a one-week history of progressive weakness, dizziness and vomiting.

She denies diarrhoea, fever, or recent foreign travel.

Her past medical history includes hypertension and systemic lupus erythematosus.

She was recently unwell with community-acquired pneumonia.

On examination she is tachycardic and a postural drop in blood pressure is noted.

She appears clinically dehydrated.

Blood tests show the following:

  • white cell count 6,
  • haemoglobin 130,
  • CRP 34,
  • Na 122,
  • K 5.7,
  • urea 7,
  • creatinine 90

Which of the following is the most likely diagnosis?

Which 4 Biochemical features are significant?

A

Addisonian crisis

Addisonian crises (also called acute adrenal insufficiency) is a life-threatening condition resulting from insufficient circulating cortisol levels.

Symptoms can be non-specific: lethargy, fever, hypotension, vomiting and diarrhoea.

Biochemical features include hyponatraemia (most classically), hyperkalaemia, hypoglycaemia and hypercalcaemia.

This can occur in patients with existing Addison’s disease (which is associated with other autoimmune conditions), or those who have suddenly stopped long-term steroids.

Triggers include increased physiological stress during infections, trauma and surgery, or simply due to missed medications.

A rarer cause is meningococcal septicaemia causing bilateral adrenal haemorrhage (Waterhouse-Friderichsen syndrome).